37 votos

La explicación intuitiva del lema de Nakayama

El lema de Nakayama establece que, dado un finamente generado $A$ -módulo $M$ y $J(A)$ el radical de Jacobson de $A$ con $I \subseteq J(A)$ algún ideal, entonces si $IM=M$ Tenemos $M=0$ .

He leído la prueba, y aunque es relativamente simple, no da mucha idea de por qué este lema debería ser cierto, por ejemplo - ¿hay alguna manera de ver cómo el hecho de que $J(A)$ es la intersección de todos los ideales máximos relacionados con el resultado?

Cualquier intuición sobre las condiciones y el resultado sería de gran ayuda.

0 votos

¿Ha leído el artículo de Wikipedia?

1 votos

@Qiaochu Yuan - Lo he mirado, pero no parece haber ninguna intuición real dada allí (al menos ninguna que yo pueda ver).

2 votos

Esta pregunta acaba de ser referenciada en MathOverflow.Net - ¡puede que quieras echar un vistazo a las respuestas! mathoverflow.net/questions/61446/

27voto

Xetius Puntos 10445

Suponga que su módulo es de longitud finita. Entonces puedes considerar en él el llamado filtración radical que organiza el módulo en forma de cebolla, con elementos del ideal máximo que empujan a los elementos del módulo más adentro de su capa inicial a una justo debajo y, además, cada capa obtenida de la anterior de esta manera.

Ahora, la condición $\mathfrak m M=M$ te dice que la capa más externa del módulo está realmente vacía: obviamente, entonces, no hay mucho en el conjunto y $M=0$ . ¡Acabamos de descubrir el lema de Nakayama!

Si su módulo es arbitrario, ocurre exactamente lo mismo.

1 votos

Me refiero al caso local, con $I$ igual al ideal máximo. El caso general no es más que una generalización natural.

1 votos

Leer un poco sobre las álgebras de caminos y sus módulos, y especialmente su interpretación como representaciones del carcaj subyacente, es una buena manera de desarrollar una intuición sobre esto, ya que te permite dibujar módulos y álgebras de una manera muy explícita, haciendo que afirmaciones como la de Nakayama sean bastante evidentes.

0 votos

Gracias por su respuesta. ¿Podría remitirme a alguna fuente donde pueda leer más sobre esta filtración radical? No encuentro nada relevante y parece que esa información podría ayudarme a ver las cosas con más claridad.

7voto

Matt Dawdy Puntos 5479

Aquí hay algo que puede o no tener sentido para ti. Sabes que todo ideal $I$ de un anillo conmutativo $R$ da lugar a un $R$ -Módulo $R/I$ Estos son precisamente los $R$ -en un generador. El $R$ -módulos de la forma $R/m$ donde $m$ es máxima son especiales entre estos. En este lenguaje, los elementos del radical de Jacobson son precisamente los que actúan trivialmente sobre todos los $R$ -módulos de la forma $R/m$ . Se deduce, por ejemplo, que si $I$ está en el radical de Jacobson, entonces no podemos tener $IM = M$ para cualquier módulo de la forma $R/m^k$ .

El lema de Nakayama afirma que una afirmación similar es cierta para todo generado finitamente $R$ -módulos. Esto debería ser razonable si se conoce, por ejemplo, la clasificación de los módulos finitamente generados $R$ -módulos cuando $R$ es un PID. La Wikipedia describe una interpretación geométrica de esto, pero no estoy lo suficientemente familiarizado con ella para decir más.

0 votos

En primer lugar, bienvenido. Esta puede ser una pregunta muy ingenua: La versión que tengo en Reid "Undergrad. Comm. Alg" se enuncia sobre un anillo local $(A,m)$ . Me preguntaba si esto es correcto: ya que $m$ es maximal, no tiene unidades. Así, para $M$ a igual $mM$ pensaría $M$ tiene que ser $0$ . Si crees que esto es digno de ser una pregunta, estaré encantado de publicarla donde puedas responderla. Gracias, saludos.

0 votos

@Andrew: se trata de un caso especial de otras afirmaciones del lema de Nakayama. Véase el artículo de Wikipedia. Si sigues confundido, puedes enviar una pregunta aparte.

6voto

Travis Puntos 517

Mis dos favoritos para la teoría de grupos en física son:

  1. Grupos de Mentira para Peatones para una introducción general a los grupos de Lie, sobre todo en un contexto de física de partículas.

  2. Química Cuántica de Levine para una introducción a la teoría de grupos en las moléculas.

5voto

Lance Wu Puntos 9

Existe una versión más primitiva y natural del lema de Nakayama:

Dejemos que $R$ sea un anillo conmutativo, y $M$ sea una entidad finitamente generada $R$ -módulo y $I\subset R$ ser un ideal. Si $IM=M$ entonces $\exists f\in I,\forall m\in M,f\cdot m=m$ .

La prueba es un poco más difícil que la versión anterior, pero esto dice más o menos "Si $IM=M$ entonces uno de los elementos de $I$ estabiliza todos los elementos de $M$ ." Y creo que esto es mucho más natural que la versión anterior. Asumiendo esta versión, entonces podemos ver que la condición de "ideal de Jacobson" sólo se utiliza para mostrar que " $1-f$ es una unidad" que lleva a " $M=0$ ".

Para completar, doy una prueba de la versión anterior aquí:

Prueba: Dejemos que $m_1,...,m_n$ sea un conjunto de generadores de $M$ . Desde $IM=M$ tenemos una matriz $A'=(a_{ij})\in \textrm{M}_{n\times n}(R)$ tal que $A'\underline{m}=\underline{m}$ donde $\underline{m}=(m_1,...,m_n)^t$ y $a_{ij}\in I,\forall i,j\leq n$ . Ahora dejemos que $A=\textrm{Id}-A'$ por lo que tenemos $A\underline{m}=\underline{0}$ .

Tenga en cuenta que $\det A\in R$ está bien definida y existe una matriz $B\in \textrm{M}_{n\times n}(R)$ s.t. $BA=AB=\det A\cdot \textrm{Id}$ La prueba se encuentra en el Corolario 9.161 del libro Advanced Modern Algebra de Joseph J. Rotman. También $\det$ conmuta con el mapa cociente $R\rightarrow R/I$ con $A\equiv \textrm{Id} \ (\textrm{mod }I)$ podemos deducir que $\det A\in 1+I$ Así que $\exists f\in I$ , s.t. $\det A=1-f$ .

Para mostrar $f$ estabiliza $M$ basta con demostrar que $f\cdot m_i=m_i,\forall i$ . $$\underline{0}=BA\underline{m}=\det A\cdot \underline{m}=(1-f)\cdot \underline{m}$$ Así que $$f\cdot \underline{m}=\underline{m}$$ El resultado es el siguiente. Q.E.D

0 votos

Creo que quieres dejar $A = \operatorname{Id} - A'$ en lugar de al revés. Buen artículo. Es la primera vez que entiendo una prueba presentada del lema.

0 votos

@feralin Tienes razón, gracias. Arreglado ahora.

0 votos

¿Puede explicarme qué significa tener $A'm = m$ ? ¿Por qué $IM = M$ implica que hay una matriz $A'$ tal que $A'm = m$ ?

4voto

Terry Lorber Puntos 1897

esta pregunta acaba de ser referenciada en MathOverflow.Net - ¡quizás quieras ver las respuestas!

http://mathoverflow.net/questions/61446/how-to-memorise-understand-nakayamas-lemma-and-its-corollaries

i-Ciencias.com

I-Ciencias es una comunidad de estudiantes y amantes de la ciencia en la que puedes resolver tus problemas y dudas.
Puedes consultar las preguntas de otros usuarios, hacer tus propias preguntas o resolver las de los demás.

Powered by:

X